If b is the midpoint of line AC, and AC = 8x-20, find BC.​

If B Is The Midpoint Of Line AC, And AC = 8x-20, Find BC.

Answers

Answer 1

The Definition Of a Midpoint says that is B is the

midpoint of segment AC, then segment AB ≅ AC.

Since segment AB ≅ segment BC, we can setup

the equation 2(3x - 1) = 8x - 20 and we can solve from here.

Start by distributing the 3 through the parenthses on the left side.

So we get 6x - 2 = 8x - 20.

Now subtract 8x from both sides to get -2x - 2 = -20.

Now add 2 to both sides and we have -2x = -18.

Finally, dividing both sides by -2, we find that x = 9.

If x = 9, segment BC must be 3(9) - 1 or 27 - 1 which is 26.


Related Questions

Armando has $25 to spend. he wants to buy 2 pounds of hamburgers at $3 per pound, a bag of buns for $2, and 5 bags of chips for $4 each. Sort each statement into True or False.

Answers

Answer:

True- She needs $3 more. She will not have enough money to buy everything. The cost is $28.

False- She has $3 left over.

Step-by-step explanation:

2 pounds of burgers for $3 each is $6

1 bag o bun equals $2

5 bags of chips for $4 each is $20

$20 + $6 +$2 = $28

She only had $25 to spent, so she doesn't have enough money and needs to 3 more dollars.

Which of the following are solutions to |x-1|= 5x + 2 ? Check all that apply.

Answers

Answer:

x = -1/6, x = -0.16

Step-by-step explanation:

3/2a-8=11 URGENT ANSWERRRRRRR

Answers

Answer:

38/3

Step-by-step explanation:

3/2a-8=11      

3/2a=19     add 8 to each side

i forget the rest of the steps smh

Answer:

38/3

Step-by-step explanation:

3/2A-8=11

3A/2=11+8

3A/2=19 CROSS MULTIPLY

3A=38

A=38/3

Find the coordinate of the midpoint of the segment formed by the given points: (7, 11) and (3, -5)

Answers

Answer:

[tex]MP = (5,3)[/tex]

Step-by-step explanation:

[tex]MP= (\frac{x_1+x_2}{2}) (\frac{y_1+y_2}{2} )\\MP= (\frac{7+3}{2}) (\frac{11+-5}{2} )\\MP= (\frac{10}{2}) (\frac{6}{2} )\\MP= (5,3)[/tex]

Find the volume of the triangular prism​

Answers

Answer:

your weight loss will help your child get through to your doctor if your child is a si and he's a doctor or 6or a child or an adult or an adult child and you can get in the way I want

HELP ASAP PLEASE HELPPPPPPP PLEASE

Answers

Answer:

5 times 3

Step-by-step explanation:

you do 5 time 3. it is 15

Why didn't the butterfly go to the dance? show your thinking!

Answers

Answer:

its math roll dont wory ill show with no math and explantion its just awnsers

Step-by-step explanation:

8 i

4 t

11 w

1 a

6 s

9 a

14 m

7 o

10 t

3 h

13 b

5 a

2 L

I couldnt get the last one but i did in the same order as it has it

To solve the question we used the criteria that the solution depends upon the sign, if negative number is greater than sign will be a minus .

if positive number is greater than sign will be a plus.

The sign while addition and subtraction depends on the larger number .

If negative number is greater than sign will be a minus .

if positive number is greater than sign will be a plus.

1.

A. -5+2=-3

B. 7+-3= 4

C. -4+-6=-10

2.

A. 1+-8=-7

B. -6+-12=-18

C. -2+9=7

3.

A. -7+6=-1

B. 5+-8=-3

C. 12+13=25

4.

A. -10+-10=-20

B. 17+-1=16

C. -11+5=-6

5.

A. 4+-9=-5

B. -7+-15=-22

C. -3+12=9

6.

A. 16+-8 = 8

B. -5+20=15

C. -6+6=0

Similarly we can do the remaining solutions.

To learn more on Number system click:

https://brainly.com/question/22046046

#SPJ2

2(10 − 6) 2 + 3(12 − 10)

Answers

I think its 22 you have to add the parentheses first and then multiply from left to right of im correct

Answer:

hope this helped you

please mark as the brainliest (ㆁωㆁ)

Given M is the midpoint of PQ with PM = 1/2x + 5 and PQ = 26, find x.
Yeah

Answers

Answer: x = 16

Step-by-step explanation:

PQ = 26

PM = 1/2 PQ (M is midpoint of PQ)

PM= 1/2x + 5 = 1/2 * 26

or x = 2(13 - 5) = 16

The value of x is 16 units.

What is a midpoint of a line ?

The midpoint of a line is the point which divide a given line in two equal parts.The formula for midpoint is Mₓ = (x₁ + x₂)/2 and Mₙ = (y₁ + y₂)/2.

(here y is replaced by n).

According to the given question Given M is the midpoint of PQ with PM = 1/2x + 5 and PQ = 26 and we have to find x.

As M is the midpoint of PQ with PM = 1/2x + 5 then MQ can also be represented by 1/2x + 5.

We know

PM + MQ = PQ

1/2x + 5 + 1/2x + 5 = 26.

x + 10 = 26.

x =16.

learn more about lines here :

https://brainly.com/question/1626339

#SPJ2

find the sum of the equation. 1000x+250x=?

Answers

Hey there!

ANSWER: [tex]1250x[/tex]EXPLANATION:

To solve, you can just add 1000 + 250 and remove x.

[tex]1000+250=1250[/tex]

Now add x.

[tex]1250x[/tex]

[tex]1000x+250x=1250x(ANSWER)[/tex]

Hope this helps!

[tex]-TestedHyperr[/tex]

Mt. Everest is 29,029 feet above sea level. The dead sea is 1,411 feet below sea level what is the difference between the elevations?

Answers

Answer:

30,440 feet

Step-by-step explanation:

You simply just add 29,029 and 1411 because if you picture it on a number line, 29,029 would be to get to zero as well as 1411 going up in elevation.

To play indoor soccer at the Sun Oaks Club, you must pay a $100 membership
fee, plus $10 for each hour you use the facilities. a) The function f represents the
cost of playing soccer at Sun Oaks depending on how many hours you play. The
function g represents the cost if the hourly rate were halved. Write each
function. *​

Answers

Given that:

Membership  feet to play indoor soccer at the Sun Oaks Club = $100

Hourly rate = $10

To find:

The function f represents the  cost of playing soccer at Sun Oaks.

Here, fixed cost is $100.

Let the number of hours of playing be t.  So, total variable cost is $10t.

Total cost = Fixed cost + Variable cost

[tex]f(t)=100+10t[/tex]

So, the required function is [tex]f(t)=100+10t[/tex].

Now, if the hourly rate were halved. Then,

Hourly rate [tex]= \$ \dfrac{10}{2} = \$ 5 [/tex]

So, total variable cost is $5t and total cost function is

[tex]g(t)=100+5t[/tex]

So, the required function is [tex]g(t)=100+5t[/tex].

Please help, image is attached! Thanks.

Answers

When something is bisected, it is cut into two equal halves.

We know that SV cut RST into two parts.

Those two parts are RSV and VST

RSV = 64 and VST = 64

RST = RSV + VST = 64 + 64 = 128

The correct answer should be 128

Answer:

I.128

Step-by-step explanation:

Since SV bisected angle RST, the angle is split in two by equal parts, therefore RSV + VST = RST

64 + 64= 128

128°= RST

A boat sailing at a speed of 35 mph set to head due west encounters a wind blowing 15 mph due south. What is the approximate resulting speed of the boat?

Answers

Answer:

38.08mph

Step-by-step explanation:

The question tells us that the speed of

Speed of boat = 35mph

Wind blowing = 15mph

We have to use the pythagorean theorem in other to find the resulting speed.

So,

Speed² = 35² + 15²

Such that

Speed = √35² + 15²

Speed = sqr(1225 + 225)

Speed = sqr(1450)

Speed = 38.08

The approximate resulting speed of the boat is therefore 38.08mph

What’s the answer 40=16-8r ?

Answers

Answer:

-3

Step-by-step explanation:

40=16-8r

Subtract 16 from both sides.

24=-8r

Now divide by -8

-3=r

Answer:

-3= r

Step-by-step explanation:

40= 16- 8r

subtract 16 from both sides

40-16= 16 - 16 -8r

24= -8r

divide both sides by -8

24/-8 = -8r/-8

-3= r

Convert 6 7/20
into a decimal

Answers

Hey there!

ANSWER: [tex]0.35[/tex]EXPLANATION:

[tex]6 \frac{7}{20}[/tex] as a decimal

The percentage for 6 7/20 is 35%. So if that was the percent, then the decimal would be 0.35.

[tex]0.35(ANSWER)[/tex]

Hope this helps!

[tex]\text {-TestedHyperr}[/tex]

(9^3)^2 Hurrrrryyyyyyyyyyyyyy

Answers

Answer: 531441

Step-by-step explanation:

Create a quadratic function f(x), other than f(x) = x^2, and write it below. Then below the function, state the domain, range, x-intercepts, y-intercept, and f(3).

Answers

Answer:

[tex]f(x)=2x^{2}-3[/tex]

[tex]D=(-\infty,\infty)[/tex]

[tex]R=[-3,\infty)[/tex]

x-intercepts: [tex](\frac{\sqrt{6}}{2},0)[/tex] and [tex](-\frac{\sqrt{6}}{2},0)[/tex]

y-intercept: (0,-3)

f(3)=15

Step-by-step explanation:

A quadratic function is any polynomial function whose greatest power is a 2, so [tex]f(x)=2x^{2}-3[/tex] is a nice example of it.

For the domain, as a rule, polynomial functions that have positive whole degrees will be defined for all real  numbers, so:

[tex]D=(-\infty,\infty)[/tex]

When talking about quadratic functions, its range will happen from the lowest point (in this case the vertex) to it's highest point (in this case infinity)

In order to find the vertex, we can use the following formula:

[tex]x=-\frac{b}{2a}[/tex]

which is used for quadratic equations in standard form:

[tex]y=ax^{2}+bx+c[/tex]

In this case a=2, b=0 and c=-3, so we get:

[tex]x=-\frac{0}{2(2)}=0[/tex]

Now we know that the range will happen at x=0, so we substitute:

[tex]f(0)=2(0)^{2}-3=-3[/tex]

so the range will be:

[tex]R=[-3,\infty)[/tex]

For the x-intercepts we need to set the given function equal to zero so we get:

[tex]2x^2-3=0[/tex]

and solve for x:

[tex]2x^{2}=3[/tex]

[tex]x^{2}=\frac{3}{2}[/tex]

[tex]x=\pm \sqrt{\frac{3}{2}}[/tex]

which can be rationalized to:

[tex]x=\frac{\sqrt{6}}{2}[/tex] and [tex]x=-\frac{\sqrt{6}}{2}[/tex]

so the x-intercepts are located at:

[tex](\frac{\sqrt{6}}{2},0)[/tex] and [tex](-\frac{\sqrt{6}}{2},0)[/tex]

For the y-intercept, we need to set x=0 so we get:

[tex]f(0)=2(0)^{2}-3[/tex]

[tex]f(0)=-3[/tex]

So the y-intercept is located at:

(0,-3)

and finally, in order to get f(3) we must substitute x for 3 so we get:

[tex]f(3)=2(3)^{2}-3[/tex]

f(3)=15

6x-3 = 5x-5 any got a homie

Answers

Answer:

x= -2

Step-by-step explanation:

6x-3 = 5x-5

6x-5x=-5+3

x= -2

See my other articles for quiz and assignment in my site.

Just copy and paste "learningandassignments diy4pro" to google search engine.

Hope it helps.

It is X= -2
I hope this helps! Here is the work! I did it on my computer!

Which two fractions are equal to 2?
A .1/2
B.2/4
C.2/1
D.2/2
E.4/4

Answers

Answer:

1/2 2/4

Step-by-step explanation:

Answer:

it is C. 2/1

Step-by-step explanation:

because 1 goes into 2 twice

Two lines are intersected by a third line.

Two horizontal parallel lines are intersected by a third line. The third line intersects the top line and forms 4 angles. Labeled clockwise, from uppercase left, the angles are 1, 2, 4, 3. The third line intersects the bottom line and forms 4 angles. Labeled clockwise, from uppercase left, the angles are 5, 6, 8, 7.
If Angle2 Is-congruent-to Angle6, which must be true about Angle2?

Angle2 Is-congruent-to Angle5
Angle2 is complementary to Angle5.
mAngle2 = mAngle8
Angle2 is supplementary to Angle8.

Answers

Answer:

Angle2 is supplementary to Angle8.

If Angle 2 Is-congruent-to Angle 6, then the statement which must be true about Angle 2 is that Angle 2 is supplementary to Angle 8.

What are Congruent Angles?

This refers to the type of angle where two different angles are said to have equal measure on their corresponding sides.

With this in mind, we can see that based on the two horizontal parallel lines are intersected by a third line, there is a third line that intersects the top line and forms four angles.

Hence, if If Angle 2 Is-congruent-to Angle 6, then the statement which must be true about Angle 2 is that Angle 2 is supplementary to Angle 8 because the sum of either angles is 180 degrees.

Read more about congruent angles here:
https://brainly.com/question/1675117

how do u find the b in y=mx+b plz help

Answers

Answer:

b = y - mx

Step-by-step explanation:

Given

y = mx + b ( subtract mx from both sides )

y - mx = b

Chrysanthemum is serving iced tea and lemonade at a picnic. She has only 80 glasses in which to
serve the drinks. If x represents the number of glasses of iced tea and y represents the number of
glasses of lemonade, which equation represents the number of glasses of ice tea she can serve?

Answers

Answer:

80=x+y

Step-by-step explanation:

The maximum amount of glasses is 80; the solution cannot be more than 80; the number of glasses of lemonade and of iced tea cannot be more than 80 total.  

-5+9(r+8)
pls show me the work

Answers

This is what I got, I haven’t done this in a while. Let me know if I’m correct and if you need any help
That is the best and correct answer to this problem the work

100 is how many more times than 10

Answers

100 is 10 more times than 10, because 10 times 10 equals 100. I hope this helps!

Answer:

10 times greater, because 10x10=100

Step-by-step explanation:

There are 32 classmates that are going to the fair. Each ticket costs $19. What is the total amount the classmates spend for tickets? Use mental math.

Answers

Answer:

32 classmates and each classmate pays $19 per ticket. This is a multiplication problem. We can solve this a couple of ways:

 

  32

*19

-------

288

320

-------

608

 

Another way is to realize 19 is nearly 20, multiply 32 by 20 and then subtract 32:

 32

*20

------

640

 

640

-32

-----

608

So, the answer is $608

Answer:

$608

Step-by-step explanation:

The answer is $608 because each ticket costs $19 and 32 students are going to the fair. Therefore, it's 32 x 19 = 608.

Hope this helps!

The end points of cd are given.find the coordinates of the midpoint M. C(-4,7) and D(0,-3)

Answers

Answer:

M=(-2,2)

Step-by-step explanation:

Xm= -4 + 0/2 --> -4/2

Ym= 7 + -3/2 --> 4/2

Complete the table and find the balance A if $5000 is invested an an annual rate of 6% for 15 years and compounded n times a year​

Answers

Answer:

1: 11982.79

2: 12136.31

4: 12216.09

12: 12270.46

365: 12297.10

Step-by-step explanation:

The compound interest formula is A=P*(1+r/n)^nt

P=principal amount

r=rate

n=number of compounds per period

t=number of periods

In this situation, the principal amount is 5,000=P. The rate is 6%, so r=0.06. n is equal to the top number in the table. And this is 15 years, so t=15.

For 1, it would be A=5,000*(1+.06/1)^1*15, or A=5,000(1.06^15), or 11982.79.

For 2, it would be 5,000*(1+.06/2)^2*15, or 5,000*(1.03^30), or 12136.31.

And so forth. Good luck

HELPPPP I MARK BRAINLIEST TO CORRECT ANSWERS

Answers

Answer:

I think B option is correct ..I m 95% sure

Answer:i

the correct option is A

First we have to convert the 3 whole 8 /33 into a fraction so the method to do that is 33*3 =99 and then 99 +8 = 107 .The denominator will remain the same so we get 107 /33.Now by dividing 107 by 33 we get 3.242424

Nate wants to visit his friend max before going to the park. Nates house is located at (-2,4), while the park is located at (10,2) find the location of macs house if it’s 1/2 of the distance from Nates house to the park

Answers

Answer:

(5,1)

Step-by-step explanation:

if macs house if it’s 1/2 of the distance from Nate house to the park then half the distance of 10,2 is 5,1

The required coordinate of the max location is (4, 3).


Given that,
Nate wants to visit his friend max before going to the park. Nate's house is located at (-2,4), while the park is located at (10,2).
To find the location of macs house if it’s 1/2 of the distance from Nate's house to the park.

What is the midpoint?

The midpoint is that point the line whose distance from both the ends of the line is equal.

Here,
Max lies 1/2 the distance of Nate and Park,
Location of max is given by,
= 10 - 2 / 2 and 4 + 2 / 2
= 8 / 2 and 6 / 2
= 4 and 3
Location of max is (4, 3)

Thus, the required coordinate of the max location is (4, 3).

Learn more about midpoints here:
https://brainly.com/question/28224145
#SPJ2


Other Questions
Is there really a secret tunnel behind mt. Rushmore????? Can someone please help? This is due today, and I have NO CLUE PLEASE HELP QUICK! Analyze evidence from the lesson regarding the origins of cell compartmentalization. Then make a claim about the origin of cell compartmentalization based upon the evidence presented, stating two to three reasons for the claim. Provide evidence and specific examples that support each reason for the claim. Provide a conclusion to the short essay. There are 10 boys and 15 girls. What is the probability of picking double girls Selected current year company information follows: Net income $ 15,953 Net sales 712,855 Total liabilities, beginning-year 83,932 Total liabilities, end-of-year 103,201 Total stockholders' equity, beginning-year 198,935 Total stockholders' equity, end-of-year 121,851 The return on total assets is: (-4.5)to the power of 0 Find distance AB.A(3, 2) and B(9,7) Goods that are considered to be needs tend to be elastic when the price changes. inelastic when the price changes. elastic when the supply changes. inelastic when the supply changes. Which statement best compares and contrasts lipids and carbohydrates?HURRY PLEASEE I HAVE TWO MINUTES JUST THE LETTER AWNSER examples of alliteration in boewulf 3^-4=????? HURRRRYYYYYYYYYYYYYYYYY Read the fragment below and then choose the correct revision of that fragment.6. The politician, whom I used to like. (1 point)On television, the politician whom I used to like.The politician in the recent election whom I used to like.Catering to everyone's wishes, the politician I used to like.The politician, whom I used to like, made many promises. What other theme is addressed in the passage? Which graph shows a polynomial function of an odd degree? F(x)=-5x+8What value of x would make f(x)=23?Need Answers ASAP What is the name of the sword that Beowulf is gifted? un adjetivo de lapiz how the edges of the Pacific plate and the North American plate converge. During the summer, the height of the water in the pool decreased by 5 inches each week due to evaporation. What is the change in the height of the water over an 8 week period? WHO EVER ANSWERS FIRST GETS BRAINLIEST I REALLY NEED HELP!!!simplify... (3/8)^-2(1/33/8)^3(1/3)^4